AIMO 2015 Q3

Algebra Level 4

A selection of 3 whatsits, 7 doovers and 1 thingy cost a total of $329. A selection of 4 whatsits, 10 doovers and 1 thingy cost $441. What is the total cost, in dollars, of 1 whatsit, 1 doover and 1 thingy?


The answer is 105.

This section requires Javascript.
You are seeing this because something didn't load right. We suggest you, (a) try refreshing the page, (b) enabling javascript if it is disabled on your browser and, finally, (c) loading the non-javascript version of this page . We're sorry about the hassle.

2 solutions

Letting w , d , t w,d,t represent the number of whatsits, doovers and thinghys, respectively, we have that

  • (i) 3 w + 7 d + t = 329 3w + 7d + t = 329 and

  • (ii) 4 w + 10 d + t = 441. 4w + 10d + t = 441.

Now as we have three variables and only two (linearly independent) equations there will be multiple positive integer solutions for ( w , d , t ) . (w,d,t). However, this does not rule out the possibility of finding a unique value for w + d + t . w + d + t. To this end, we need to find values a , b a,b such that

a ( 3 w + 7 d + t ) + b ( 4 w + 10 d + t ) = w + d + t ( 3 a + 4 b ) w + ( 7 a + 10 b ) d + ( a + b ) t = w + d + t . a*(3w + 7d + t) + b*(4w + 10d + t) = w + d + t \Longrightarrow (3a + 4b)w + (7a + 10b)d + (a + b)t = w + d + t.

Equating like coefficients, we require that 3 a + 4 b = 1 , 7 a + 10 b = 1 3a + 4b = 1, 7a + 10b = 1 and a + b = 1. a + b = 1.

Then 3 a + 4 b 3 ( a + b ) = 1 3 = 2 b = 2 , a = 3 , 3a + 4b - 3(a + b) = 1 - 3 = -2 \Longrightarrow b = -2, a = 3, which also satisfies 7 a + 10 b = 1. 7a + 10b = 1.

Thus w + d + t = 3 329 2 441 = 105 . w + d + t = 3*329 - 2*441 = \boxed{105}.

Possible positive integer solutions for ( w , d , t ) (w,d,t) include ( 22 , 30 , 53 ) , ( 52 , 20 , 33 ) (22,30,53), (52,20,33) and ( 82 , 10 , 13 ) . (82,10,13). (There are in fact 34 34 such solutions, ranging from ( 1 , 37 , 67 ) (1,37,67) to ( 100 , 4 , 1 ) (100,4,1) .)

Isnt it better to multiply 1 by 3 and 2 by 4 and simply subtract them

Kaustubh Miglani - 5 years, 5 months ago

Log in to reply

But that would give us 3 ( 3 w + 7 d + t ) 4 ( 4 w + 10 d + t ) = 7 w 19 d t 3(3w + 7d + t) - 4(4w + 10d + t) = -7w - 19d - t , which doesn't really help.

My method ends up multiplying (i) by 3 and (ii) by 2 and then subtracting, which does result in w + d + t w + d + t .

Brian Charlesworth - 5 years, 5 months ago

Log in to reply

Yeah,sorry I meant to say the same thing only i just typed wrongly.Iam such a fool

Kaustubh Miglani - 5 years, 5 months ago

Log in to reply

@Kaustubh Miglani No worries. :)

Brian Charlesworth - 5 years, 5 months ago
谦艺 伍
Aug 10, 2016

3 w + 7 d + t = 329 3w + 7d + t = 329 ..........(1)

4 w + 10 d + t = 441 4w + 10d + t = 441 ..........(2)

(2) - (1): w + 3 d = 112 w + 3d = 112 ..........(3)

(1) - 2*(3): w + d + t = 105 w + d + t = 105

0 pending reports

×

Problem Loading...

Note Loading...

Set Loading...